Flugbahn des bergab geworfenen Projektils

Ich bringe mir Mechanik bei und wollte ein Problem lösen, bei dem es darum geht, den optimalen Winkel zu bestimmen, um ein Projektil zu werfen, wenn man auf einem Hügel steht, um eine maximale Reichweite zu erzielen. Meine Antwort scheint fast plausibel, bis auf einen Begriff, der, um plausibel zu sein, sein Vorzeichen wechseln muss. Aber ich kann kein Loch in meiner Argumentation finden.

Problem: Ich stehe auf einem geraden, abfallenden Hügel und möchte einen Stein für maximale Reichweite werfen. Der Hügel ist von der Horizontalen nach unten geneigt φ . Welcher Winkel θ über der Horizontalen soll ich es anwerfen?

Meine Lösung:

  1. Verwenden Sie dazu Koordinaten X ist parallel zum Hügel

  2. Lassen a = φ + θ (das heißt, der Winkel über dem Boden, auf den ich werfe)

  3. Dann initial v Ist v X = cos a , v j = Sünde a (Normalisierung der Einheiten, um Konstanten zu entfernen)

  4. Erdbeschleunigung ist dann A X = k Sünde φ , A j = k cos φ (Schwerkraft ist in y-Richtung). Nehmen Sie zur Vereinfachung der Berechnungen an k = 2 (Die Antwort gilt für jeden Wert der Schwerkraft, also auf dem Mond wie auf der Erde)

  5. Wir wollen das Alpha finden, das maximiert S X zu der Zeit, die macht S j = 0 . Finden Sie zuerst die Zeit, die macht S j = 0 ; nenne es t.

  6. S j = T Sünde a T 2 cos φ . Mit der quadratischen Formel S j = 0 bei T = 0 oder T = Sünde a cos φ .

  7. Jetzt finden S X bei diesem T . Durch Einsetzen und Verwenden von grundlegender Algebra und Trig erhalten wir S X = Sünde a cos a Sünde 2 a bräunen φ . (Das macht Sinn; der erste Term ist maximal π / 4 , wie wir es von Symmetrie erwarten würden. Der zweite Term sagt uns, dass wir unseren Wurfwinkel verringern sollten, wenn der Boden stark abfällt. Sehr plausibel.)

  8. Wenn wir phi als Konstante nehmen, möchten wir diesen Ausdruck maximieren. Ein wenig Kalkül und trigonometrische Identitäten bringen die Ableitung gleich cos ( 2 a ) Sünde ( 2 a ) bräunen φ , die eine Null bei hat a = π / 4 φ / 2 , oder θ = π / 4 3 φ / 2 . Hier brechen die Dinge zusammen. Der erste Begriff, π / 4 , scheint richtig. Aber der zweite Term liefert lächerliche Ergebnisse.

  9. Das Umschalten des Vorzeichens des zweiten Terms in der Alpha-Gleichung endet mit θ = π / 4 φ / 2 , was durchaus plausible Ergebnisse liefert. Aber ich kann keinen Fehler in meiner Argumentation oder Berechnung finden!

Kann jemand den fehlenden Link finden?


Erklärung der Antwort:

Wie Pygmalion festgestellt hat, ist Schritt 4 falsch. Der A j Wert ist korrekt , aber A X sollte positiv sein : zeigt den Hügel hinunter.

Die Antwort ist unabhängig von der Größe der Schwerkraft; aber es kommt auf die richtung an .

Überarbeitung der Herleitung:

7. S X = Sünde a cos a + Sünde 2 a bräunen ϕ

8. Ableitung ist cos ( 2 a ) + Sünde ( 2 a ) bräunen ϕ , mit Null bei a = π / 4 + ϕ / 2 , daher θ = π / 4 ϕ / 2 . QED.

Willkommen bei PhysSE. Bitte verwenden Sie beim nächsten Mal die mathematische Formatierung, damit x = sin theta wird X = Sünde θ Zum Beispiel.
Ein Punkt: Ich behaupte, die Antwort ist unabhängig von der Schwerkraft, genauso wie auf Mond und Mars. Aber was ist mit "Rückwärtsgravitation" (vom Boden wegdrücken)? Das würde sicherlich die Antwort ändern. Das löst die Dinge nicht, aber es kann ein guter Anfang sein.
Warum nicht das horizontale vertikale Koordinatensystem (z X , j Komponenten) und der Trajektorie entsprechen j ( T ) , X ( T ) zum Gefälle von bräunen φ oder j ( T ) = X ( T ) bräunen φ ?
@S.RobertJames: Ich denke, dass die Antwort vom Verhältnis der Anfangsgeschwindigkeit zur Schwerkraft abhängt.
Okay, es ist unabhängig von G . Meine Formel ist R = 2 u 2 G cos φ ( cos θ Sünde ( φ + θ ) ) , maximal bei θ = π 2 φ 2 . Welches ist deines?
@Manishearth Maximum muss sein π / 4 + etwas. Überprüfen Sie die Grenze φ = 0
@Pygmalion: Ja, ist es π 4 (Ich habe es falsch von meinem Scribblepad kopiert), aber ich bekomme immer noch ein Minus. Sind Sie sicher, dass Sie den Ball nach unten werfen ? In diesem Fall gibt es ein Plus.
@Manisharth Vielleicht ja. Das Abwärtsrechnen scheint eher ein intuitives Problem zu sein als das Gegenteil. S.RobertJames soll uns sagen...

Antworten (1)

Soweit ich sehe, A j = k cos ( φ ) < 0 , Aber A X = k Sünde ( φ ) > 0 ! Zumindest wenn man das Projektil nach unten wirft...

Ich glaube, sie sind beide < 0 . A ist aufgrund der Schwerkraft, und die Schwerkraft ist gerade nach unten, im Uhrzeigersinn gedreht ϕ aufgrund der Koordinatendrehung. Rotierend [ 0 1 ] von ϕ , 0 < ϕ < π / 2 ergibt beide Komponenten negativ.
@S.RobertJames Wenn Sie nach unten werfen , ist die maximale Entfernung größer , das liegt daran A j ist kleiner u A X ist positiv (hilft Ihnen). Vergessen Sie das Drehen, zeichnen Sie das Bild.
Bitte erklären Sie, warum Sie sich fühlen A X ist positiv (hilft). Obwohl ich Ihren Standpunkt intuitiv sehe (die Schwerkraft hilft mir, einen Hügel hinunterzuwerfen), ergibt das Drehen der Koordinaten (wie in meinem vorherigen Kommentar beschrieben) kein negatives (verletzendes) Ergebnis. A X ?
Ich lese das 0 ! als „nullfaktoriell“. Hat mich dazu gebracht "Warte, was?". Dann habe ich mir die Frage noch einmal angesehen, um sicherzustellen, dass es sich nicht um eine rekursive Frage handelt (bei der es mehrere Bounces gibt). ;-)
Verstanden: Die Koordinatenachsen drehen sich im Uhrzeigersinn. Die Vektoren von den Anfangskoordinaten rotieren gegen den Uhrzeigersinn. Somit, A X dreht sich positiv! Ausgezeichnet, danke Pygmalion. Du bist Nummer e \ich π + 1 ! (Ich ziehe meinen Hut vor Manisharth).
Ja, S. Robert James Manchmal ist es eine gute Idee, das Problem zu skizzieren. :) Wenn dies Ihre Frage beantwortet, möchten Sie sie vielleicht akzeptieren, indem Sie auf die grüne Schaltfläche unter der Frage klicken :)
S. Robert James: Sehr erfreut. :)
@Manishearth Ich habe nicht ganz verstanden 0! Kommentar. War es auf mich gerichtet?
@Pygmalion Ich habe es auch in meiner Auszeichnung verwendet :-)
Ups, ich habe Hölle statt Hilfe geschrieben. Aber ich denke, du hast die richtige Botschaft. :)